LSAT and Law School Admissions Forum

Get expert LSAT preparation and law school admissions advice from PowerScore Test Preparation.

 Administrator
PowerScore Staff
  • PowerScore Staff
  • Posts: 8916
  • Joined: Feb 02, 2011
|
#26306
Complete Question Explanation

Flaw in the Reasoning. The correct answer choice is (C)

The stimulus states that Michaela’s play will be performed by the theater this season, and we know that critical acclaim is one of the main factors considered in the selection of plays to perform. However, this does not mean that critical acclaim is the only factor considered, nor that critical acclaim is required in order for a play to be selected. The conclusion mistakenly concludes that Michaela must be a critically acclaimed playwright. In other words, the author concludes that critical acclaim is necessary for a play to be chosen. There is nothing in the premises to substantiate such a strong claim.

Answer Choice (A): This answer choice describes a Mistaken Reversal. The author did not make this mistake. If the author had made this mistake, she would have said: “You must be critically acclaimed to be selected. Michaela is critically acclaimed. Therefore, her play was selected.” The argument in this stimulus never made this mistake.

Answer Choice (B): This answer choice describes an error in causal reasoning. There was no causal reasoning present in this stimulus, so the answer choice is incorrect.

Answer Choice (C): This is the correct answer choice. The author did confuse a main factor for a factor that was necessary for selection. The facts here do not indicate critical acclaim was ever required for a play to be selected.

Answer Choice (D): The only source in this argument is the production director. There is no reason to suspect that the production director is unreliable in describing one of the main factors in selecting plays to perform.

Answer Choice (E): Once again, this answer choice describes a causal error. Specifically, this answer choice is suggesting that the author confused a cause for an effect. Since there was no causal reasoning in this stimulus, this answer choice does not accurately describe the error in the stimulus and is therefore incorrect.
 axanasoltan
  • Posts: 1
  • Joined: Nov 07, 2016
|
#30350
Dear Powerscore Tutor,

I am confused why Answer Choice (A) is wrong and C is the correct answer. In the explanation above, you have included that A is mistaken reversal. However, it does make sense to me how it is a mistaken reversal. Can you please expand on this more? Thank you for your time.
 Adam Tyson
PowerScore Staff
  • PowerScore Staff
  • Posts: 5153
  • Joined: Apr 14, 2011
|
#30406
Thanks for asking, axanasoltan! This does look a little like a Mistaken Reversal, doesn't it? It's close, but not quite there, and here's why: the author never said that "if critically acclaimed, then selected". If that was the premise, then this would indeed be a classic MR and answer A would be a winner.

Instead, though, our author made a less powerful claim, one that was not at all conditional. Clark said only that critical acclaim is a main factor in making that decision, not that it was, by itself, sufficient. That takes any conditional analysis of this question off the table and knocks answer A out of the running. Close, but no cigar!

In order for us to use conditional analyses for these questions we have to be sure conditionality is present, and we can often do that by paraphrasing the argument as an if...then argument, as I tried to do with this up above. Since this argument doesn't fit into that simple conditional framework without altering the meaning or the original claim, we have to try another approach and reject that one.

Look again and see if that makes sense to you. Then, keep coming back here for more questions. We're happy to help!

Get the most out of your LSAT Prep Plus subscription.

Analyze and track your performance with our Testing and Analytics Package.